Ejercicio 9.b Última pregunta resolución

Ejercicio 9.b Última pregunta resolución

de Juan Pedro Maestrone Vazquez -
Número de respuestas: 2

Intenté llevar a cabo el problema 9.b tomando el S´ dado en la última pregunta y resulta que en la velocidad me sobra un término.

Adjunto imagen de lo que hice, si alguien me puede informar qué está mal se lo agradezco.


43 palabras

Adjunto Pregunta final ej. 9.b.jfif
En respuesta a Juan Pedro Maestrone Vazquez

Re: Ejercicio 9.b Última pregunta resolución

de Enzo Spera -

Hola, tenes un error al derivar en el sistema móvil. En el sistema móvil el versor \hat{e}_{x} está fijo por lo que su derivada es cero.

Entonces:

\vec{v}\;'=\dot{x}\hat{e}_{x}

Así mismo, a tu velocidad angular le falta un término, \vec{\omega}=(\dot{\psi}+\dot{\theta})\hat{k} esto es porque \dot{\psi} es la velocidad angular de la barra OA, pero P se mueve sobre la barra AB la cual también tiene velocidad angular \dot{\theta} .

Entonces al hacer:

\vec{\omega}\wedge\vec{r}\;'=x(\dot{\psi}+\dot{\theta})\hat{e}_{\theta}


Saludos, Enzo

101 palabras

En respuesta a Enzo Spera

Re: Ejercicio 9.b Última pregunta resolución

de Enzo Spera -

Hola, revisando la respuesta que escribí ayer ahora veo que tu pregunta pasa por la solución publicada hace unos días. En esa solución se toma el sistema de referencia con origen en A solidario a la barra OA.

Disculpa ayer entendí mal tu pregunta. Con este sistema de referencia, el punto P solo rota lo que rota el ángulo \theta por lo que en este referencial \vec{\omega}=\dot{\theta}\hat{k}. Entonces no te esta sobrando el término x\dot{\psi}\hat{e}_{\theta} porque la velocidad relativa es:

\vec{v}\;'=\dot{x}\hat{e}_x+x\dot{\theta}\hat{e}_{\theta}

Tabién está explicado por Esteban Mato en el tema: Ejercicio 9 parte b

Saludos, Enzo.

118 palabras